Finding & Classifying Local/Absolute Extrema of f(x,y)

  • Thread starter Thread starter kekido
  • Start date Start date
  • Tags Tags
    Extrema
Click For Summary
The discussion focuses on finding and classifying the local and absolute extrema of the function f(x,y)=x-xy within the defined region D. The critical point identified is (0,1), where the Hessian matrix indicates that the quadratic form is inconclusive, suggesting that this point is a saddle point. The user seeks guidance on determining the absolute extrema within the specified region, noting challenges in converting the function to polar coordinates for analysis. The conversion to polar coordinates complicates finding extrema due to the nature of the function and the constraints of the region. The thread highlights the need for further clarification on absolute extrema in the given context.
kekido
Messages
18
Reaction score
0

Homework Statement



Find and classify the local and absolute extrema of the function
f(x,y)=x-xy over the region
D={(x,y)|x^2+y^2\leq1 and x+y\geq0}

Homework Equations





The Attempt at a Solution


Critical points are where the first derivative (gradient) is 0.
\nablaf=(1-y, -x)=0
So critical point a=(0,1)

In order to classify the critical point, find the Hessian matrix of f at a:
H=\left(\begin{array}{cc}0&-1\\-1&0\end{array}\right)
Then the quadratic form is:
Q(x,y)=Hk\cdotk=-2xy
Q(a)=Q(0,1)=0

Which means the test is inconclusive? I.e., the critical point a is a saddle point, which is neither local maximum or minimum. Am I right?


Also, how do you find the absolute extreme of the function on the region aforementioned?

I tried to convert x^2+y^2<=1 to polar coordinates, which gives
r^2\cos^2\eta+r^2\sin^2\eta\leq1
r^2\leq1
0&lt;r\leq1
However, this doesn't help much as the original function converted to polar coordinates is not straightforward to find its extrema given the domain of r and theta.
 
Physics news on Phys.org
Question: A clock's minute hand has length 4 and its hour hand has length 3. What is the distance between the tips at the moment when it is increasing most rapidly?(Putnam Exam Question) Answer: Making assumption that both the hands moves at constant angular velocities, the answer is ## \sqrt{7} .## But don't you think this assumption is somewhat doubtful and wrong?

Similar threads

  • · Replies 11 ·
Replies
11
Views
2K
  • · Replies 8 ·
Replies
8
Views
3K
  • · Replies 7 ·
Replies
7
Views
3K
Replies
11
Views
2K
  • · Replies 6 ·
Replies
6
Views
2K
  • · Replies 20 ·
Replies
20
Views
1K
  • · Replies 11 ·
Replies
11
Views
2K
Replies
10
Views
1K
  • · Replies 8 ·
Replies
8
Views
2K
Replies
5
Views
2K